subject
Health, 12.03.2022 08:40 my7butterflies

A 31-year-old man incurs an injury in an industrial accident, with a steel H-beam striking the right side of his face, crushing the zygomatic arch and damaging the globe of the right eye. He undergoes plastic repair of bone and soft tissues and an attempt is made to salvage the eye. A month later, he has decreased visual acuity in both eyes, with choroidal thickening seen on funduscopic examination. The right eye is enucleated and shows prominent CD4 lymphocyte and macrophage infiltration of the uvea. By which of the following immunologic mechanisms has this condition most likely developed?
A. Breakdown of T-cell anergy
B. Polyclonal lymphocyte activation
C. Release of sequestered antigens
D. Cross-reactivity with myocardial antigens
E. Failure of T-cell mediated suppression

ansver
Answers: 1

Another question on Health

question
Health, 22.06.2019 12:20
The dietary guidelines for americans recommend (a) avoiding smoking. (b) engaging in aerobic exercise for 60 minutes at least five days a week. (c) making all of your grain choices whole grains. (d) increasing your intake of fruits and vegetables.
Answers: 3
question
Health, 22.06.2019 22:30
What kind of wound occurred when the skin scrapes against a hard surface and tiny blood vessels break in the outer layer of skin
Answers: 1
question
Health, 23.06.2019 00:30
George’s daughter was born with a hole in her heart. he met up with a group of individuals of questionable character who agreed to deliver to george a healthy heart that could be a transplant for his daughter for a sum of $20,000. the group george negotiated with failed to deliver the heart. george tried to sue when he did not receive the heart but was surprised when the courts would not enforce the agreement.can george force performance of this contract? why or why not? (10 points)will he be able to recover the $20,000 that he gave for the heart? why or why not? (10 points)
Answers: 2
question
Health, 23.06.2019 14:30
What should i do when a baby has gas at night?
Answers: 2
You know the right answer?
A 31-year-old man incurs an injury in an industrial accident, with a steel H-beam striking the right...
Questions
question
Mathematics, 13.07.2020 20:01
question
Mathematics, 13.07.2020 20:01
Questions on the website: 13722367